Which one of the following, if true, would be the most reasonable response for the author of passage B to make to the...

avif on June 23, 2020

A vs. B

I limited down the answers to A and B. I am a bit confused though on what the last argument in the passage is. I thought that the main argument was that the rich pay less taxes legally and illegally, and then it gave an example how a legal way to get out of taxes was. If that is part of the argument I thought that only A can address that by saying that there is not necessarily a benefit to switch (even though that's not a great argument as many wouldn't try to get out of taxes then). B on the other hand doesn't address the first part at all which I thought was the main part of the final argument. Please explain. Thanks

Reply
Create a free account to read and take part in forum discussions.

Already have an account? log in

Annie on June 23, 2020

Hi @avif,

This question asks you to pick the answer choice which provides the most reasonable response for Author B to make to Author A over Author A's last paragraph.

So, to start, turn to Author A's last paragraph. In this paragraph, Author A argues that flat taxes are in fact progressive, as they exempt a certain amount of income from any taxation, and then all taxation above that is taxed with a single rate. Author A further argues that in progressive-tax countries there are high incentives for wealthy individuals to avoid paying taxes at their correct rate which means they take steps to avoid paying at that rate by utilizing the complex tax code. In the end, they often end up paying as much as they would under a flat-tax regime.

Turning to the answer choices:

(A) is incorrect because it doesn't counter the argument in Passage A. The author in passage A is arguing that flat taxes are better than progressive ones, for the reasons explained above. The author in passage B argues that that is wrong. This answer choice doesn't support the idea that progressive taxes are better, rather it just says there's a way in which flat taxes could be equally as bad.

(B) is correct because it both counters the argument in passage A and supports the argument in passage B. This statement directly counters Author A's contention that the reason wealthy people pay less taxes in a progressive system is because they take advantage of the complexity of the tax code. Instead, Author B is saying that they are supposed to be getting these deductions, and thus that this is not a flaw of a progressive code.